Exercises:Rings and Modules - 2016 - 1/Problem 1

From Maths
< Exercises:Rings and Modules - 2016 - 1
Revision as of 16:05, 19 October 2016 by Alec (Talk | contribs) (Questions written)

(diff) ← Older revision | Latest revision (diff) | Newer revision → (diff)
Jump to: navigation, search

Problems

Problem 1

Part A

Let [ilmath]R[/ilmath] be a u-ring. Fix an [ilmath]a\in R[/ilmath] and define a homomorphism:

  • [ilmath]\varphi_a:R[T]\rightarrow R[/ilmath] by [ilmath]\varphi_a:P(T)\mapsto P(a)[/ilmath] - evaluation at [ilmath]a[/ilmath].

By restriction of scalars every [ilmath]\varphi_a[/ilmath] gives the target [ilmath]R[/ilmath] the structure of an [ilmath]R[T][/ilmath]-module, which we denote [ilmath]R_a[/ilmath].

Show that for [ilmath]a,b\in R[/ilmath] that:

  • there is an [ilmath]R[T][/ilmath]-module isomorphism between [ilmath]R_a[/ilmath] and [ilmath]R_b[/ilmath]

if and only if

  • [ilmath]a=b[/ilmath]
Solution

Part B

Let [ilmath]M[/ilmath] be an [ilmath]R[/ilmath]-module. Show that there is a surjection from a free [ilmath]R[/ilmath]-module onto [ilmath]M[/ilmath].

Solution

Part C

Show that the [ilmath]\mathbb{Z} [/ilmath]-module, [ilmath]\mathbb{Q} [/ilmath], is not free.

Solution

Notes

References